[Boards: 3 / a / aco / adv / an / asp / b / bant / biz / c / can / cgl / ck / cm / co / cock / d / diy / e / fa / fap / fit / fitlit / g / gd / gif / h / hc / his / hm / hr / i / ic / int / jp / k / lgbt / lit / m / mlp / mlpol / mo / mtv / mu / n / news / o / out / outsoc / p / po / pol / qa / qst / r / r9k / s / s4s / sci / soc / sp / spa / t / tg / toy / trash / trv / tv / u / v / vg / vint / vip / vp / vr / w / wg / wsg / wsr / x / y ] [Search | Free Show | Home]

Post Math Challenges/Questions

This is a blue board which means that it's for everybody (Safe For Work content only). If you see any adult content, please report it.

Thread replies: 75
Thread images: 10

Post Math Challenges/Questions. I'm bored and I have no more tears, post math questions so I can try to solve.
>>
File: 1481037733098.jpg (3MB, 3500x3500px) Image search: [Google]
1481037733098.jpg
3MB, 3500x3500px
>>8809565
Last time I posted this one, /sci/ spurged out.
>>
>>8809565
sum all the prime numbers less than 2 million
>>
what about solving some introductory physics
>>
>>8809608
try looking in your 10th grade physics textbook.
>>
>>8809565
Here's a quickie: derive the conservation of energy from first principles
>>
Let [math]f: [0,\infty) \to \mathbb R[/math] a continuous surjection. Prove that f assumes each value infinitely many times.
>>
>>8810926
Wow, that one is cool. Immediately tried to find a counterexample, because my intuition said it's wrong.

Here:

[math]
\begin{align*}
x^{x^{x^{.^{.^{.}}}}} &= 2 \\
x^2 &= 2 \\
x &= \sqrt{2}
\end{align*}
[/math]

and also

[math]
\begin{align*}
x^{x^{x^{.^{.^{.}}}}} &= 4 \\
x^4 &= 4 \\
x &= \sqrt[4]{4} \\
x &= \sqrt{2}
\end{align*}
[/math]

but then:

[math]
2 = \sqrt{2}^{\sqrt{2}^{\sqrt{2}^{.^{.^{.}}}}} = 4
[/math]

Whats wrong here?
>>
>>8810980
put that last one in a calculator
>>
Given a positive integer n, let M(n) be the largest integer m such that:

mC(n-1) > (m-1)C(n)

Evaluate the limit as n approaches infinity of M(n)/n
>>
>>8810980
>Whats wrong here?

That you never proved that any of those things make sense and you are just basically saying
"Algebra is magic".
>>
Take a 3 digit number A, reverse its digits to get B.
If A > B then X = A - B
If A < B then X = B - A

Reverse the digits of X (If its a 2 digit number put a zero on the front i.e. 099) to get Y

Show that X + Y always equals the same number regardless of A
>>
>>8810926
log(x) is a counterexample, right?
>>
>>8810926
let there be a value [math]y \in \mathbb{R} [/math] which [math]f[/math] assumes only finitely many times, let [math]x_0[/math] be the largest point such that [math]f(x_0) = y [/math]. now [math]f( (x_0,\infty) )[/math] is a connected set lying in [math]\mathbb{R} \setminus \{ y \}[/math] and WLOG is therefore fully contained in [math] (y,\infty)[/math]. this implies that [math]f([0,x_0]) [/math], a compact set, contains all [math](-\infty,y)[/math]. contradiction.

amirite?
>>
>>8811154
I also first thought of log(x), but it's domain is (0,infty) not [0,infty)

>>8811112
No, no, no, it was supposed to be a math challenge. One of them is correct, the challenge is to find out why the other one is not. (Yes you can use a calculator but where is the fun?)
>>
>>8811165
Looks correct, well done
>>
>>8811139
That one's pretty cool. Only times X + Y don't equal 1,089 are when the first and last of the 3 numbers are the same
>>
>>8811171
Oh yeah, I forgot that detail in the question

It's a simple enough question to state that my normie Dad even enjoyed hearing the solution
>>
>>8811139
A = a100 + b10 + c10
B = c100 + b10 + a
A > B iff a > c
A < B iff a < c
suppose a > c
X = A - B = (a-c)100 + (c-a)
but (c-a) is not a digit so we have
X = (a-c)100 - (a-c)
X = (a-c)99

Now there are 9 possibilities
X = 099,198,297,396,495,594,693,792,891
Y = 990,891,792,693,594,495,396,297,198

Then notice that:

X + Y = k*99 + (11-k)99 = 99k + 11*99 - 99k = 11*99

Nice.
I am taking an elementary course this semester. Did I do well daddy?
>>
>>8811180
Exactly what I did to solve it

Find a 9 digit number such that the first two digits taken together are a multiple of 2, the first 3 digits are a multiple of 3 and so on, so the whole 9 digit number is a multiple of 9. There's only one solution
>>
>>8810980
If [math] x^{x^{x^{.^{.^{.}}}}} = 2 [/math] then [math] x = \sqrt{2} [/math] is a correct statement.

If [math] x^{x^{x^{.^{.^{.}}}}} = 4 [/math] then [math] x = \sqrt{2} [/math] is also a correct statement.


If [math] x^{x^{x^{.^{.^{.}}}}} = 2 [/math] AND [math] x^{x^{x^{.^{.^{.}}}}} = 4 [/math] then [math]2 = \sqrt{2}^{\sqrt{2}^{\sqrt{2}^{.^{.^{.}}}}} = 4 [/math] is also a correct statement.


The only "issue" is that [math]2 = \sqrt{2}^{\sqrt{2}^{\sqrt{2}^{.^{.^{.}}}}} = 4 [/math] is wrong but none of the calculations show otherwise.
>>
>>8811184
Let this number be
qwertyuio
(, means or)
If the first two digits are a multiple of 2 then
w = 0,2,4,6,8
if the first 3 digits are a multiple of 3 then
q + w + e = 3k
r10 + e = 4k
t=0,5
y=0,2,4,6,8
r + t + y = 3k
u = y = i = 0
q+w+e+r+t+y+u+i+o = 9k

r=1,e=6
w=0
q=6,3

THE ANSWER IS:
306450963

Not gonna lie there m8. I didn't even use the answers I originally wrote out. But after I had like 6 digits secured I started trial and erroring.
Great problem m8.
>>
>>8811231
Ehh that defo works, but I forgot to mention that you have you use all the digits 1,2,3,4,5,6,7,8,9. That is, no 0 allowed. With 0 allowed there are many solutions but without it there is only one

I also remember another digits based problem but I've forgotten the details so I'll find that in a bit
>>
>>8811247
Found it again

Find a 5 digit number A such that when you reverse its digits to get a new number B = 4 * A
>>
>>8811261
Fuck should be a 4 digit number
>>
>>8811247
Ah. I don't sure if that makes it easier or harder. With that then it is guaranteed that the 5th digit is 5, for example.
I am now busier so I'll see if I revisit this later.

Here is one my prof left for homework yeasterday:
Find digits x,y,z such that 5, 9 and 11 divide 2x1642y032z
>>
>>8811265
28164280320
>>
>>8811265
>>8811289
28164230325 works too.

The equations are:
z = 0 OR z = 5
x + y + z = 7 OR x + y + z = 16 OR x + y + z = 25
x = y + z OR x + 11 = y + z
>>
Let [math]X, Y \in \mathcal M_{n-1,1}(\mathbb R)[/math] and [eqn]M = \begin{pmatrix} 0 & X^T \\ Y & 0_{n-1}\end{pmatrix}[/eqn]
Give a necessary and sufficient condition for M to be diagonalizable.
>>
File: 1457673217805.gif (2MB, 400x290px) Image search: [Google]
1457673217805.gif
2MB, 400x290px
You have four numbers: 2,5,8,9.
You have to use these numbers to get 52, using the four basic operations (+,-,*,/).
You can only use each number once and you can use as many brackets as you want.
>>
>>8809568
Answer is 0.867.
>>
>>8809565
My favourite one from a calculus class is:

Given the longitude and latitude of two cities and the radius of the earth, find the distance between the two cities.

Pretty simple but fun application!
>>
File: continuity proof.png (10KB, 796x45px) Image search: [Google]
continuity proof.png
10KB, 796x45px
>>
>>8811955
>Give a necessary and sufficient condition for M to be diagonalizable.

A necessary and sufficient condition is that M has n linearly independent eigenvectors.
>>
>>8811997
8*(9-5/2)
>>
>>8809565
HAHAHAHAHAHAHAHAHA yeah we're turning into third world nations. Murica got its niggers and massive corruption /mind control/indoctrination and stupid fat people. And we've got muslims to fuck our shit up
>>
>>8811036
(3+sqrt(5))/2
>>
>>8812028
kek
why not just cut to the chase

>A necessary and sufficient condition for M to be diagonalizable is that M is diagonalizable
>>
>>8812050
>>8812028
wew lads, a condition on X and Y (that can be checked in linear time if you're going to tell me that the condition was also a necessary and sufficient condition on X and Y)

Here's another one:
Let [math](a_n)[/math] and [math](b_n)[/math] be two real sequences such that [math](a_n + b_n) \rightarrow 0 [/math] and [math](e^{a_n} + e^{b_n}) \to 2[/math]. Prove that both sequences converge.
>>
>>8812650
Suppose that [math](a_n)[/math] doesn't converge then one of two cases must happen:

Case 1: [math](a_n)[/math] has an unbounded subsequence [math](a_{n_k})[/math].
Without loss of generality suppose that [math] \lim_{k \to \infty} a_{n_k} = \infty [/math]. Then we must have [math] \lim_{k \to \infty} b_{n_k} = - \infty [/math] but then [math]\lim_{k \to \infty} (e^{a_{n_k}} + e^{b_{n_k}}) = \infty [/math] which is a contradiction.

Case 2: [math](a_n)[/math] has a subsequence [math](a_{n_k})[/math] that converges against a [math] c \neq 0 [/math].
Then [math] \lim_{k \to \infty} b_{n_k} = -c [/math] and [math]\lim_{k \to \infty} (e^{a_{n_k}} + e^{b_{n_k}}) = e^c + e^{-c} [/math] but [math] e^c + e^{-c} \neq 2 [/math] for [math]c \neq 0 [/math] so we have another contradiction.


Therefore out assumption must be wrong and [math](a_n)[/math] converges to [math]0 [/math]. Then [math](b_n)[/math] must naturally converge to [math]0 [/math] too.
>>
Represent 0.dddddddd... As a fraction where d exists in {1,2,3,4,5,6,7,8,9,0}
>>
>>8813078
d/9
>>
>>8809565
x and y are reals.
1) Study f(x, y) = (x + y, xy).

2) We have the system (u = x + y, v = xy). Find x and as functions of u and v.
>>
>>8813084
Prove it
>>
>>8809565
This one is ez.

Let S be Shalaquandra's ass and B the blank space between her ass and the radius

r = r
>>
>>8810980
You have a radius of convergence for the infinite power tower. It is clear that the square root of 2 is in the radius of convergence, but 4 is not.
>>
>>8813141
[math]
0.ddd... = d\cdot \sum_{k=1}^{\infty}\left(\frac{1}{10}\right)^k = d\cdot \left(\frac{1}{1-\frac{1}{10}} -1 \right) = \frac{d}{9}
[/math]
>>
File: 1491397187909-v.jpg (49KB, 500x375px) Image search: [Google]
1491397187909-v.jpg
49KB, 500x375px
>>8809570
That thread was stupid
>>
File: primes.png (2KB, 262x116px) Image search: [Google]
primes.png
2KB, 262x116px
>>8809570
>>8813225
Where exactly is the difficulty supposed to be?
>>
File: Miami.png (12KB, 516x375px) Image search: [Google]
Miami.png
12KB, 516x375px
Requires calculus
>>
>>8813259
[math] \int_0^{10} \left( \int_0^{10 cos(x/10)} y dy \right) dx = 25(5sin(2)+10)[/math]
>>
>>8813287
I forgot to divide by the area of the quarter of circle
>>
>>8812713
Great, that works.
Here's one in algebra.
Let M be a matrix with real coefficients. What is, in terms of M, the least number of coefficients you need to change to make it invertible ?
>>
File: 1272740561363.gif (4KB, 466x291px) Image search: [Google]
1272740561363.gif
4KB, 466x291px
what is the total area of the sum of all the grey circles?
>>
>>8813297
HA what kind of nonsens is that! that has no aplication in the real world! dont waste your time!
>>
>>8813297
Infinity.
>>
>>8813291
wat
>>
>>8813290
>>8813287
not quite
>>
>>8813259
For people having a hard time
[math]
\bar{y} = (\frac{M_{xz}}{M}) = \frac{\int\int y dA}{\int\int 1 dA}= \frac{\int_{x=-10}^{x=10}\int_{y=0}^{y=\pm\sqrt{100-x^{2}}} ydydx}{\frac{\pi r^{2}}{2}}
[/math]
>>
File: continuity proof2.png (18KB, 810x86px) Image search: [Google]
continuity proof2.png
18KB, 810x86px
To all the analysts:
How can I express delta as a function of epsilon without including c? I can't seem to figure it out. Any hints?
>>
>>8813826
[eqn] \delta_\varepsilon = \sup_x |g'(x)| \varepsilon [/eqn]

You can do this for all [math]C^1[/math] functions. Just look at the fundamental theorem of Calculus.
>>
>>8814040
are you saying all C^1 functions are uniformly continuous? because that's not true
>>
>>8814046
The ones with [math] \sup_x |g'(x)| < \infty [/math] are.
>>
>>8814058
right so nowhere near all C^1 functions... not even x^2
>>
>>8814040
It's actually
[eqn] \sup_x |g'(x)| \delta_\varepsilon < \varepsilon [/eqn]

[eqn] \left| g(x) - g(c) \right| = \left| \int_c^x g'(t) dt \right| \leq \|g'\|_{\infty} |x - c| < \varepsilon [/eqn]
>>
>>8813291
if M is an n*n matrix, then you need n changes. or more precisely, you need k changes where k is the algebraic multiplicity of 0 as the eigenvalue of M.
>>
>>8814075
I would have said geometric multiplicity,(that is, n-rk(M)), maybe that's what you meant ? Take the matrix [math]M = \begin{pmatrix}0 & 1 \\ 0 & 0 \end{pmatrix}[/math]. You need only change the bottom right coefficient to make it invertible but 0 has algebraic multiplicity 2.
In any case I expected a proof.

>>8813335
Given an n*n matrix, would you agree that you can make it invertible by changing all the coefficients ? (just replace each coefficient by the correspondent coefficient in the identity matrix). Now i'm asking the minimum number of changes you need to do this.
>>
>>8814122
Can't you just take any value [math]a [/math] that is not in the spectrum and then subtract [math]a [/math] from the diagonal?
>>
>>8814147
Yup, that means changing n coefficients (one for each on the diagonal), but you can do better than that
>>
>>8814122
no, I meant algebraic, but it's wrong, should be geometric. I didn't realize that changing one entry could change more than one eigenvalues
>>
>>8813297
[math].86997R^2[/math]
>>
>>8809568
I get 1.758
>>
>>8813297
I think a good first question might be "why is this construction even possible ?"
>>
File: 1491428232889.gif (2MB, 400x229px) Image search: [Google]
1491428232889.gif
2MB, 400x229px
>>8815404
This is what Wolfram tells me
>>
>>8812046
Show the werk son.
Thread posts: 75
Thread images: 10


[Boards: 3 / a / aco / adv / an / asp / b / bant / biz / c / can / cgl / ck / cm / co / cock / d / diy / e / fa / fap / fit / fitlit / g / gd / gif / h / hc / his / hm / hr / i / ic / int / jp / k / lgbt / lit / m / mlp / mlpol / mo / mtv / mu / n / news / o / out / outsoc / p / po / pol / qa / qst / r / r9k / s / s4s / sci / soc / sp / spa / t / tg / toy / trash / trv / tv / u / v / vg / vint / vip / vp / vr / w / wg / wsg / wsr / x / y] [Search | Top | Home]

I'm aware that Imgur.com will stop allowing adult images since 15th of May. I'm taking actions to backup as much data as possible.
Read more on this topic here - https://archived.moe/talk/thread/1694/


If you need a post removed click on it's [Report] button and follow the instruction.
DMCA Content Takedown via dmca.com
All images are hosted on imgur.com.
If you like this website please support us by donating with Bitcoins at 16mKtbZiwW52BLkibtCr8jUg2KVUMTxVQ5
All trademarks and copyrights on this page are owned by their respective parties.
Images uploaded are the responsibility of the Poster. Comments are owned by the Poster.
This is a 4chan archive - all of the content originated from that site.
This means that RandomArchive shows their content, archived.
If you need information for a Poster - contact them.